Does there exist the limit at infinity of a function $f:[1,+infty)to[0,+infty)$ such that...












3












$begingroup$


Let $f:[1,+infty)to[0,+infty)$ be a function such that $int_{1}^{+infty}f(x)dx$ is convergent. I have 3 questions as follows:




  1. Does $f(x)$ have limit at infinity?

  2. If the answer of question (1) is yes, then is $lim_{xtoinfty}f(x)=0$ true?

  3. If the answers of questions (1) and (2) are NEGATIVE, What can we say about their answers, if $f$ is a continuous function?


I found that the most function under this condition have limit at infinity, also their limit equal to zero. Moreover, I tried to apply the definition of improper integral
$$int_{1}^{infty}f(x)dx=lim_{btoinfty}int_{1}^{b}f(x)dx$$



to find Contradictionو but I could not arrive at the aim.










share|cite|improve this question











$endgroup$



closed as off-topic by RRL, A. Pongrácz, Cesareo, Adrian Keister, metamorphy Jan 19 at 15:17


This question appears to be off-topic. The users who voted to close gave this specific reason:


  • "This question is missing context or other details: Please provide additional context, which ideally explains why the question is relevant to you and our community. Some forms of context include: background and motivation, relevant definitions, source, possible strategies, your current progress, why the question is interesting or important, etc." – RRL, A. Pongrácz, Cesareo, Adrian Keister, metamorphy

If this question can be reworded to fit the rules in the help center, please edit the question.





















    3












    $begingroup$


    Let $f:[1,+infty)to[0,+infty)$ be a function such that $int_{1}^{+infty}f(x)dx$ is convergent. I have 3 questions as follows:




    1. Does $f(x)$ have limit at infinity?

    2. If the answer of question (1) is yes, then is $lim_{xtoinfty}f(x)=0$ true?

    3. If the answers of questions (1) and (2) are NEGATIVE, What can we say about their answers, if $f$ is a continuous function?


    I found that the most function under this condition have limit at infinity, also their limit equal to zero. Moreover, I tried to apply the definition of improper integral
    $$int_{1}^{infty}f(x)dx=lim_{btoinfty}int_{1}^{b}f(x)dx$$



    to find Contradictionو but I could not arrive at the aim.










    share|cite|improve this question











    $endgroup$



    closed as off-topic by RRL, A. Pongrácz, Cesareo, Adrian Keister, metamorphy Jan 19 at 15:17


    This question appears to be off-topic. The users who voted to close gave this specific reason:


    • "This question is missing context or other details: Please provide additional context, which ideally explains why the question is relevant to you and our community. Some forms of context include: background and motivation, relevant definitions, source, possible strategies, your current progress, why the question is interesting or important, etc." – RRL, A. Pongrácz, Cesareo, Adrian Keister, metamorphy

    If this question can be reworded to fit the rules in the help center, please edit the question.



















      3












      3








      3





      $begingroup$


      Let $f:[1,+infty)to[0,+infty)$ be a function such that $int_{1}^{+infty}f(x)dx$ is convergent. I have 3 questions as follows:




      1. Does $f(x)$ have limit at infinity?

      2. If the answer of question (1) is yes, then is $lim_{xtoinfty}f(x)=0$ true?

      3. If the answers of questions (1) and (2) are NEGATIVE, What can we say about their answers, if $f$ is a continuous function?


      I found that the most function under this condition have limit at infinity, also their limit equal to zero. Moreover, I tried to apply the definition of improper integral
      $$int_{1}^{infty}f(x)dx=lim_{btoinfty}int_{1}^{b}f(x)dx$$



      to find Contradictionو but I could not arrive at the aim.










      share|cite|improve this question











      $endgroup$




      Let $f:[1,+infty)to[0,+infty)$ be a function such that $int_{1}^{+infty}f(x)dx$ is convergent. I have 3 questions as follows:




      1. Does $f(x)$ have limit at infinity?

      2. If the answer of question (1) is yes, then is $lim_{xtoinfty}f(x)=0$ true?

      3. If the answers of questions (1) and (2) are NEGATIVE, What can we say about their answers, if $f$ is a continuous function?


      I found that the most function under this condition have limit at infinity, also their limit equal to zero. Moreover, I tried to apply the definition of improper integral
      $$int_{1}^{infty}f(x)dx=lim_{btoinfty}int_{1}^{b}f(x)dx$$



      to find Contradictionو but I could not arrive at the aim.







      calculus improper-integrals






      share|cite|improve this question















      share|cite|improve this question













      share|cite|improve this question




      share|cite|improve this question








      edited Jan 19 at 17:31







      soodehMehboodi

















      asked Jan 18 at 16:37









      soodehMehboodisoodehMehboodi

      65848




      65848




      closed as off-topic by RRL, A. Pongrácz, Cesareo, Adrian Keister, metamorphy Jan 19 at 15:17


      This question appears to be off-topic. The users who voted to close gave this specific reason:


      • "This question is missing context or other details: Please provide additional context, which ideally explains why the question is relevant to you and our community. Some forms of context include: background and motivation, relevant definitions, source, possible strategies, your current progress, why the question is interesting or important, etc." – RRL, A. Pongrácz, Cesareo, Adrian Keister, metamorphy

      If this question can be reworded to fit the rules in the help center, please edit the question.







      closed as off-topic by RRL, A. Pongrácz, Cesareo, Adrian Keister, metamorphy Jan 19 at 15:17


      This question appears to be off-topic. The users who voted to close gave this specific reason:


      • "This question is missing context or other details: Please provide additional context, which ideally explains why the question is relevant to you and our community. Some forms of context include: background and motivation, relevant definitions, source, possible strategies, your current progress, why the question is interesting or important, etc." – RRL, A. Pongrácz, Cesareo, Adrian Keister, metamorphy

      If this question can be reworded to fit the rules in the help center, please edit the question.






















          2 Answers
          2






          active

          oldest

          votes


















          3












          $begingroup$

          Excellent question! Turns out the answer to (1) is negative. See this post. However, if $f$ does have a limit at $infty$, it turns out it is actually $0$, as you suspected. This is an interesting exercise, try solving it :)






          share|cite|improve this answer









          $endgroup$













          • $begingroup$
            Are you sure about your sentence "However, if $f$ does have a limit at $infty$, it turns out it is actually $0$". If yes, would you prove that please.
            $endgroup$
            – soodehMehboodi
            Jan 18 at 18:33








          • 1




            $begingroup$
            Yes. Here's a sketch of the proof: if the limit is $+infty$, $f$ is greater than $1$ from some point $M$. But the integral of $1$ from $M$ to $infty$ diverges, so by comparison test so does the integral of $f$, contradiction. If the limit is $-infty$, examine $-f$ and you'll get a contradiction from what we proved earlier. I'll leave the finite case to you, as it is very similar.
            $endgroup$
            – Yuval Gat
            Jan 18 at 18:42










          • $begingroup$
            @YuvalGat: The limit being $pm infty$ is far from the only option. Did you mean to say that if the limit is $A>0$, then $f(x)>A/2$ from some point $M$?
            $endgroup$
            – Hans Lundmark
            Jan 18 at 18:51










          • $begingroup$
            @HansLundmark Of course the infinities aren’t the only options! I left the finite case as an exercise, as I stated in my original comment. Perhaps you missed that last sentence there :)
            $endgroup$
            – Yuval Gat
            Jan 18 at 18:54










          • $begingroup$
            @YuvalGat: Indeed I did! Sorry if I spoiled the exercise... ;-)
            $endgroup$
            – Hans Lundmark
            Jan 19 at 9:19



















          2












          $begingroup$

          There are simple counterexamples to your first question. For instance, the indicator function $f = 1_{mathbb Z}$ does not have a limit at infinity, but is integrable (Riemann and Lesbegue) with $int_1^infty f, dx = 0$.



          The case when $f$ is continuous does not hold either, but here the counterexamples are technical. See the answers to this question.






          share|cite|improve this answer









          $endgroup$




















            2 Answers
            2






            active

            oldest

            votes








            2 Answers
            2






            active

            oldest

            votes









            active

            oldest

            votes






            active

            oldest

            votes









            3












            $begingroup$

            Excellent question! Turns out the answer to (1) is negative. See this post. However, if $f$ does have a limit at $infty$, it turns out it is actually $0$, as you suspected. This is an interesting exercise, try solving it :)






            share|cite|improve this answer









            $endgroup$













            • $begingroup$
              Are you sure about your sentence "However, if $f$ does have a limit at $infty$, it turns out it is actually $0$". If yes, would you prove that please.
              $endgroup$
              – soodehMehboodi
              Jan 18 at 18:33








            • 1




              $begingroup$
              Yes. Here's a sketch of the proof: if the limit is $+infty$, $f$ is greater than $1$ from some point $M$. But the integral of $1$ from $M$ to $infty$ diverges, so by comparison test so does the integral of $f$, contradiction. If the limit is $-infty$, examine $-f$ and you'll get a contradiction from what we proved earlier. I'll leave the finite case to you, as it is very similar.
              $endgroup$
              – Yuval Gat
              Jan 18 at 18:42










            • $begingroup$
              @YuvalGat: The limit being $pm infty$ is far from the only option. Did you mean to say that if the limit is $A>0$, then $f(x)>A/2$ from some point $M$?
              $endgroup$
              – Hans Lundmark
              Jan 18 at 18:51










            • $begingroup$
              @HansLundmark Of course the infinities aren’t the only options! I left the finite case as an exercise, as I stated in my original comment. Perhaps you missed that last sentence there :)
              $endgroup$
              – Yuval Gat
              Jan 18 at 18:54










            • $begingroup$
              @YuvalGat: Indeed I did! Sorry if I spoiled the exercise... ;-)
              $endgroup$
              – Hans Lundmark
              Jan 19 at 9:19
















            3












            $begingroup$

            Excellent question! Turns out the answer to (1) is negative. See this post. However, if $f$ does have a limit at $infty$, it turns out it is actually $0$, as you suspected. This is an interesting exercise, try solving it :)






            share|cite|improve this answer









            $endgroup$













            • $begingroup$
              Are you sure about your sentence "However, if $f$ does have a limit at $infty$, it turns out it is actually $0$". If yes, would you prove that please.
              $endgroup$
              – soodehMehboodi
              Jan 18 at 18:33








            • 1




              $begingroup$
              Yes. Here's a sketch of the proof: if the limit is $+infty$, $f$ is greater than $1$ from some point $M$. But the integral of $1$ from $M$ to $infty$ diverges, so by comparison test so does the integral of $f$, contradiction. If the limit is $-infty$, examine $-f$ and you'll get a contradiction from what we proved earlier. I'll leave the finite case to you, as it is very similar.
              $endgroup$
              – Yuval Gat
              Jan 18 at 18:42










            • $begingroup$
              @YuvalGat: The limit being $pm infty$ is far from the only option. Did you mean to say that if the limit is $A>0$, then $f(x)>A/2$ from some point $M$?
              $endgroup$
              – Hans Lundmark
              Jan 18 at 18:51










            • $begingroup$
              @HansLundmark Of course the infinities aren’t the only options! I left the finite case as an exercise, as I stated in my original comment. Perhaps you missed that last sentence there :)
              $endgroup$
              – Yuval Gat
              Jan 18 at 18:54










            • $begingroup$
              @YuvalGat: Indeed I did! Sorry if I spoiled the exercise... ;-)
              $endgroup$
              – Hans Lundmark
              Jan 19 at 9:19














            3












            3








            3





            $begingroup$

            Excellent question! Turns out the answer to (1) is negative. See this post. However, if $f$ does have a limit at $infty$, it turns out it is actually $0$, as you suspected. This is an interesting exercise, try solving it :)






            share|cite|improve this answer









            $endgroup$



            Excellent question! Turns out the answer to (1) is negative. See this post. However, if $f$ does have a limit at $infty$, it turns out it is actually $0$, as you suspected. This is an interesting exercise, try solving it :)







            share|cite|improve this answer












            share|cite|improve this answer



            share|cite|improve this answer










            answered Jan 18 at 16:40









            Yuval GatYuval Gat

            1,0441213




            1,0441213












            • $begingroup$
              Are you sure about your sentence "However, if $f$ does have a limit at $infty$, it turns out it is actually $0$". If yes, would you prove that please.
              $endgroup$
              – soodehMehboodi
              Jan 18 at 18:33








            • 1




              $begingroup$
              Yes. Here's a sketch of the proof: if the limit is $+infty$, $f$ is greater than $1$ from some point $M$. But the integral of $1$ from $M$ to $infty$ diverges, so by comparison test so does the integral of $f$, contradiction. If the limit is $-infty$, examine $-f$ and you'll get a contradiction from what we proved earlier. I'll leave the finite case to you, as it is very similar.
              $endgroup$
              – Yuval Gat
              Jan 18 at 18:42










            • $begingroup$
              @YuvalGat: The limit being $pm infty$ is far from the only option. Did you mean to say that if the limit is $A>0$, then $f(x)>A/2$ from some point $M$?
              $endgroup$
              – Hans Lundmark
              Jan 18 at 18:51










            • $begingroup$
              @HansLundmark Of course the infinities aren’t the only options! I left the finite case as an exercise, as I stated in my original comment. Perhaps you missed that last sentence there :)
              $endgroup$
              – Yuval Gat
              Jan 18 at 18:54










            • $begingroup$
              @YuvalGat: Indeed I did! Sorry if I spoiled the exercise... ;-)
              $endgroup$
              – Hans Lundmark
              Jan 19 at 9:19


















            • $begingroup$
              Are you sure about your sentence "However, if $f$ does have a limit at $infty$, it turns out it is actually $0$". If yes, would you prove that please.
              $endgroup$
              – soodehMehboodi
              Jan 18 at 18:33








            • 1




              $begingroup$
              Yes. Here's a sketch of the proof: if the limit is $+infty$, $f$ is greater than $1$ from some point $M$. But the integral of $1$ from $M$ to $infty$ diverges, so by comparison test so does the integral of $f$, contradiction. If the limit is $-infty$, examine $-f$ and you'll get a contradiction from what we proved earlier. I'll leave the finite case to you, as it is very similar.
              $endgroup$
              – Yuval Gat
              Jan 18 at 18:42










            • $begingroup$
              @YuvalGat: The limit being $pm infty$ is far from the only option. Did you mean to say that if the limit is $A>0$, then $f(x)>A/2$ from some point $M$?
              $endgroup$
              – Hans Lundmark
              Jan 18 at 18:51










            • $begingroup$
              @HansLundmark Of course the infinities aren’t the only options! I left the finite case as an exercise, as I stated in my original comment. Perhaps you missed that last sentence there :)
              $endgroup$
              – Yuval Gat
              Jan 18 at 18:54










            • $begingroup$
              @YuvalGat: Indeed I did! Sorry if I spoiled the exercise... ;-)
              $endgroup$
              – Hans Lundmark
              Jan 19 at 9:19
















            $begingroup$
            Are you sure about your sentence "However, if $f$ does have a limit at $infty$, it turns out it is actually $0$". If yes, would you prove that please.
            $endgroup$
            – soodehMehboodi
            Jan 18 at 18:33






            $begingroup$
            Are you sure about your sentence "However, if $f$ does have a limit at $infty$, it turns out it is actually $0$". If yes, would you prove that please.
            $endgroup$
            – soodehMehboodi
            Jan 18 at 18:33






            1




            1




            $begingroup$
            Yes. Here's a sketch of the proof: if the limit is $+infty$, $f$ is greater than $1$ from some point $M$. But the integral of $1$ from $M$ to $infty$ diverges, so by comparison test so does the integral of $f$, contradiction. If the limit is $-infty$, examine $-f$ and you'll get a contradiction from what we proved earlier. I'll leave the finite case to you, as it is very similar.
            $endgroup$
            – Yuval Gat
            Jan 18 at 18:42




            $begingroup$
            Yes. Here's a sketch of the proof: if the limit is $+infty$, $f$ is greater than $1$ from some point $M$. But the integral of $1$ from $M$ to $infty$ diverges, so by comparison test so does the integral of $f$, contradiction. If the limit is $-infty$, examine $-f$ and you'll get a contradiction from what we proved earlier. I'll leave the finite case to you, as it is very similar.
            $endgroup$
            – Yuval Gat
            Jan 18 at 18:42












            $begingroup$
            @YuvalGat: The limit being $pm infty$ is far from the only option. Did you mean to say that if the limit is $A>0$, then $f(x)>A/2$ from some point $M$?
            $endgroup$
            – Hans Lundmark
            Jan 18 at 18:51




            $begingroup$
            @YuvalGat: The limit being $pm infty$ is far from the only option. Did you mean to say that if the limit is $A>0$, then $f(x)>A/2$ from some point $M$?
            $endgroup$
            – Hans Lundmark
            Jan 18 at 18:51












            $begingroup$
            @HansLundmark Of course the infinities aren’t the only options! I left the finite case as an exercise, as I stated in my original comment. Perhaps you missed that last sentence there :)
            $endgroup$
            – Yuval Gat
            Jan 18 at 18:54




            $begingroup$
            @HansLundmark Of course the infinities aren’t the only options! I left the finite case as an exercise, as I stated in my original comment. Perhaps you missed that last sentence there :)
            $endgroup$
            – Yuval Gat
            Jan 18 at 18:54












            $begingroup$
            @YuvalGat: Indeed I did! Sorry if I spoiled the exercise... ;-)
            $endgroup$
            – Hans Lundmark
            Jan 19 at 9:19




            $begingroup$
            @YuvalGat: Indeed I did! Sorry if I spoiled the exercise... ;-)
            $endgroup$
            – Hans Lundmark
            Jan 19 at 9:19











            2












            $begingroup$

            There are simple counterexamples to your first question. For instance, the indicator function $f = 1_{mathbb Z}$ does not have a limit at infinity, but is integrable (Riemann and Lesbegue) with $int_1^infty f, dx = 0$.



            The case when $f$ is continuous does not hold either, but here the counterexamples are technical. See the answers to this question.






            share|cite|improve this answer









            $endgroup$


















              2












              $begingroup$

              There are simple counterexamples to your first question. For instance, the indicator function $f = 1_{mathbb Z}$ does not have a limit at infinity, but is integrable (Riemann and Lesbegue) with $int_1^infty f, dx = 0$.



              The case when $f$ is continuous does not hold either, but here the counterexamples are technical. See the answers to this question.






              share|cite|improve this answer









              $endgroup$
















                2












                2








                2





                $begingroup$

                There are simple counterexamples to your first question. For instance, the indicator function $f = 1_{mathbb Z}$ does not have a limit at infinity, but is integrable (Riemann and Lesbegue) with $int_1^infty f, dx = 0$.



                The case when $f$ is continuous does not hold either, but here the counterexamples are technical. See the answers to this question.






                share|cite|improve this answer









                $endgroup$



                There are simple counterexamples to your first question. For instance, the indicator function $f = 1_{mathbb Z}$ does not have a limit at infinity, but is integrable (Riemann and Lesbegue) with $int_1^infty f, dx = 0$.



                The case when $f$ is continuous does not hold either, but here the counterexamples are technical. See the answers to this question.







                share|cite|improve this answer












                share|cite|improve this answer



                share|cite|improve this answer










                answered Jan 18 at 17:39









                o.h.o.h.

                6917




                6917















                    Popular posts from this blog

                    Human spaceflight

                    Can not write log (Is /dev/pts mounted?) - openpty in Ubuntu-on-Windows?

                    File:DeusFollowingSea.jpg